2
$\begingroup$

Suppose that a polynomial $p(z)$ of degree $n$ does not assume the value $w$ for $|z|<1$, that is $p(z)\neq w$ for $|z|<1.$ Show that $p(z)-\dfrac{(1-e^{i\psi})}{n}zp^{\prime}(z)\neq w$ for $|z|<1,\psi\in\mathbb{R}.$

The polar derivative of a polynomial $p(z)$ is defined as $$ D_\alpha p(z):=np(z)+(\alpha -z)p^{\prime}(z) \qquad \alpha\in\mathbb{C}. $$ If all the zeros of $p(z)$ lie inside a circular region $\mathcal{C},$ then by Laguerre's Separation theorem]1 the zero $w$ of $D_\alpha p(z)$ and the point $\alpha$ cannot lie both outside $\mathcal{C}.$ Above result follows by applying Laguerre's theorem to $p(z)-w.$

I am looking for a proof which is independent of Laguerre's theorem.

$\endgroup$
3
  • 4
    $\begingroup$ This is not true: take p(z)=z$, $\psi=\pi$, $w=1$. $\endgroup$ Mar 23, 2015 at 3:07
  • 1
    $\begingroup$ <strike>Huh? What is the value omited by $p(z)=z$ ?</strike> Sorry, I had only read the title $\endgroup$ Mar 23, 2015 at 5:41
  • 1
    $\begingroup$ @ Alexandre: There was a typo error in expression $p(z)-\dfrac{(1-e^{i\psi})}{n}zp^{\prime}(z).$ $\endgroup$
    – Suhail
    Mar 23, 2015 at 13:23

1 Answer 1

5
$\begingroup$

We can assume that $w=0$ (replace $p$ by $q=p-w$). Then $p=c\prod (z-a_j)$ with $|a_j|\ge 1$. Since $$ \frac{p'}{p} =\sum_{j=1}^n \frac{1}{z-a_j} , $$ we want to show that $$ 1 - \frac{1-e^{i\psi}}{n} \sum_{j=1}^n \frac{z}{z-a_j} $$ does not take the value $0$ on $|z|<1$, or, equivalently, the equation $$ \frac{1}{1-e^{i\psi}} = \frac{1}{n} \sum_{j=1}^n \frac{1}{1-a_j/z} \quad\quad\quad (1) $$ can not be solved with a $|z|<1$.

The map $z\mapsto 1/z$ maps the circle of radius $1$ with center $z_0=1$ to the vertical line $L$ given by $\textrm{Re}\, z=1/2$. Now $1-e^{i\psi}$ is on this circle while the $1-a_j/z$ are outside (recall that $|a_j|\ge 1$, $|z|<1$). Thus what we are trying to do in (1) is to satisfy $$ u = \frac{1}{n} \sum_{j=1}^n v_j $$ with $u$ on $L$ and all $v_j$'s on one side of $L$. Clearly this is impossible.

$\endgroup$

Your Answer

By clicking “Post Your Answer”, you agree to our terms of service and acknowledge you have read our privacy policy.

Not the answer you're looking for? Browse other questions tagged or ask your own question.